Allison owns a food truck that sells tacos and burritos. She only has enough ingredients to make 98 tacos or burritos.Write an inequality that could represent the possible values for the number of tacos sold, tt, and the number of burritos sold, bb, that would satisfy the constraint

Answers

Answer 1

Answer:

98=49bb+49tt

Explain:

so u have the same amount of both


Related Questions

If x=4 and y=5 and z=-2 what is the answer for this equation: 3x + 1

Answers

Answer:

13

Step-by-step explanation:

We only need to pay attention to the x. We multiply 3 and 4 to get 12. Then we add 1 to get 13.

Answer:

13

Step-by-step explanation:

3x + 1

Let x = 4

3*4 +1

Multiply

12+1

13

A sector with a radius of 15cm has a central angle measure of 8pi/5 (in radians)

Answers

Answer:

565.49 cm2

Step-by-step explanation:

The area of a circle is given by:

Area = pi * r^2

And the whole area represents a central angle of 2pi.

Then, to find the area of this sector, we just need to use a rule of three:

area of pi * r^2 -> central angle of 2pi

area of x -> central angle of 8pi/5

x * 2pi = pi * r^2 * 8pi/5

2x = r^2 * 8pi/5

x = r^2 * 4pi/5 = 15^2 * 4pi/5 = 565.49 cm2

Answer:180pi cm^2

Step-by-step explanation:

Cuz I got it right

5y + 3 > −7y + 13 Multi-step linear inequalities

Answers

Answer:

y > [tex]\frac{5}{6}[/tex]

Step-by-step explanation:

Given

5y + 3 > - 7y + 13 ( add 7y to both sides )

12y + 3 > 13 ( subtract 3 from both sides )

12y > 10 ( divide both sides by 12 )

y > [tex]\frac{10}{12}[/tex] , that is

y > [tex]\frac{5}{6}[/tex]

Which line is a slop that is positive

Answers

Answer: A

Step-by-step explanation:

Answer:

the correct answer i believe is A. :)

Really stuck on this please help!

Answers

Answer:

4) BC=30.14

11)XZY≈47.5°

Step-by-step explanation:

4) we use cosine law because we have two length known and one angle known

BC²=AB²+AC²-2ABcos55°

BC²=23²+20²-2*23*20cos55°=908.6

BC=√908.6= 30.14 cm

11) we use cosinee law because three length known

cosXZY=(22²+20²-17²)/(2×22×20)

cosXZY=0.676

XZY^= cos^-1 0.676

XZY^=47.5°

How do you calculate the interquartile range of a dot plot or data set?

Answers

Answer:

The IQR describes the middle 50% of values when ordered from lowest to highest. To find the interquartile range (IQR), ​first find the median (middle value) of the lower and upper half of the data. These values are quartile 1 (Q1) and quartile 3 (Q3). The IQR is the difference between Q3 and Q1.

Step-by-step explanation:

hope this helps

correct me if this is wrong

How do you solve for all values of x by factoring

Answers

Answer:

Just follow the guideline given below.

Step-by-step explanation:

To solve values of x by factoring, you need to follow these steps:

1. Move all terms to one side of the equation, usually the left, using addition or subtraction.

2. Factor the equation completely.

3. Set each factor equal to zero, and solve it.

4. List each solution from 3 as a solution to the original equation.

You usually end up with an answer like x =.. or x = ... and so on.

What is the standard notation of 415 km?

Answers

Answer:

The answer is 415 because it is already in standard notation.

What is the ratio of the volume of Cylinder A to the volume of Cylinder B? If cylinder A has a radius of 3 and a height of 4 and cylinder B has a radius of 4 and a height of 3

Answers

Answer:

3:4

Step-by-step explanation:

Volume of cylinder A = V= π[tex]r^{2}[/tex]h = r,3 h,4 = [tex]\pi[/tex]36

Volume of cylinder B = V= π[tex]r^{2}[/tex]h = r,4 h,3 = [tex]\pi[/tex]48

Ratio of the volume = [tex]\frac{36}{48}[/tex]

= [tex]\frac{18}{24}[/tex]

= [tex]\frac{9}{12}[/tex]

= [tex]\frac{3}{4}[/tex]

= 3:4

Hope it helps!

Mark as brainliest!

Please help I don’t get it!!!
( I will mark brainliest!)

Answers

Answer:

15/4 OR 3 whole number 3/4

Step-by-step explanation:

Y
U(7,6)
T(3,4)
X
0
(k, k+8)
The figure shows a straight line passing through the points T(3, 4) and U(7,6).
(a) Find the gradient of TU.
(b) Find the equation of the line TU.
(c) A point (k, k+ 8) lies on the line TU produced. Find the value of k.​

Answers

Answer:

a) 1/2

b) y = 0.5x + 2,5

c) k = -3

Step-by-step explanation:

Given are points T(3, 4) and U(7,6).

To find the gradient of TU, you look at the incline in value in the y-direction, divided by the incline in value in the x-diretion.

(Uy- Ty) / (Ux- Tx)

substitute the values given and you get:

(6-4) / (7-3)

2 / 4

This gives you the answer on question a) The gradient of TU = 1/2

Any line is of the form y = ax + b with a = the gradient found in answer a) an b is the the value on the y-coordinate on the y axis, where the line TU intersects with the y-axis.

The whole next part is my attempt to explain to you something about getting the answer for b. Sorry if it is so long, but please take the time to read the explanation, and I hope it will help you to understand what is going on.

The gradient of TU = 1/2 which means that if you start on any point on the line, and then you go 2 units to the right and 1 up you will end upon another point on the line. That is because the quotient is the same. 2/4 = 1/2 = 0,5/1

The last quotient, 0,5/1 , also has the resulting value of 0,5.

This means that if you start on any point on the line, and then you go 1 unit to the right and 0.5 up, you will be on another point on the line.

ATTENTION : This means also, that if you start on any point on the line TU, and then you go 1 unit to the LEFT and 0.5 DOWN, you will be on another point on the line TU. If you understand this, than you have learned something important!

We need to know where the line TU intersects with the y-axis, and now we can use this to find that value?

Since point T(3,4) lies 3 units to the right of the y-axis, we need to go 3 units to the LEFT. But remember the incline of 0.5/1 ... For every unit you go to the left you need to go 0,5 unit DOWN to remain on the line.

If you go 3vunits to the left, how many units do you need to go down?

3 * 0.5 units which is 1.5.

Point T has an y coordinate of 4 so ... what will the y coordinate be if you go 3 units to the left?

4 - (1.5) = 2.5

So now finally we can write down the answer to question b).

y = 0.5x + 2,5

c) (k, k+ 8) lies on the line TU. So substitute that in y = 0.5x + 2,5

inplace of x you write kinplace of y you write k+8

y = 0.5 * ( x ) + 2,5

k+8 = 0.5 * ( k + 8 ) + 2,5

k+8 = 0.5*k + 4 + 2,5

Bring all k values to the left of the equality sign, and all number values to the right.

k - 0.5*k = 4 + 2,5 - 8

0.5*k = 6,5 - 8

0.5*k = -1,5

multiply left and right of the = sign by 2 and you get your answer on question c).

k = -3

4. In the hockey season, Chris had 150 shots on goal. She scored on 22% of her
shots. How many goals did she score?

Answers

Answer:

She scored on 33 of them

Step-by-step explanation:

You take 150 and multiply it by .22. which is 22%

A group of 6th-grade students ran the 50-yard dash. Their times are
shown in the line plot below. What was the median time for the 50-yard
dash?
Time on 50-yard Dash
(in seconds)

Answers

Answer:

the median time for the 50 yard dash is 8 seconds

Step-by-step explanation:

hope this helps :)

To divide 1.89 ÷ 0.9, we need to change it. Choose the correct problem below to show what the “new” look would be.
A. 189 ÷ 9
B. 18.9 ÷ 0.9
C. 1.89 ÷ 9
D. 18.9 ÷ 9

Answers

Answer:

D. 18.9 ÷ 9

Step-by-step explanation:

we need to divide

1.89 ÷ 0.9 but write it in different form

so ,we need to eliminate decimal from 0.9

as 0.9*10 = 9

thus,we multiply both  1.89 and  0.9 with 10, then we will have

(1.89*10) ÷ (0.9*10)

=> 18.9 ÷ 9

Thus, based on above calculation new look would be D. 18.9 ÷ 9

what is the volume of this container ​

Answers

Answer:

1468 in³

Step-by-step explanation:

First get get the total volume of the rectangular prism:

length l = 10 in

width w = 15 in

height h = 10 in

diagonal d = 20.6155281 in

total surface area Stot = 800 in²

lateral surface area Slat = 500 in²

top surface area Stop = 150 in²

bottom surface area Sbot = 150 in²

volume V = 1500 in

Now get get the volume from the missing rectangular prism:

ength l = 4 in

width w = 4 in

height h = 2 in

diagonal d = 6 in

total surface area Stot = 64 in²

lateral surface area Slat = 32 in²

top surface area Stop = 16 in²

bottom surface area Sbot = 16 in³

volume V = 32 in³

So 1500in³- 32 in³ =1468 in³

Answer:

1468

Step-by-step explanation:

10*10*15 = 1500

1500 - (4*4*2) = 1468

1500-32=1468

Therefore the answer is 1468

What are the zeros of the function f(x) = x2 + 5x + 5 written in simplest radical form?
Quadratic formula: x =
- Vb2 - 4 ac
2a

Answers

Answer: ([tex]\frac{-5 +\sqrt{5} }{2}[/tex], 0), ([tex]\frac{-5 -\sqrt{5} }{2}[/tex], 0)

Step-by-step explanation: Let me know if you need an explanation.

Graph the points ( 0, 2 ), ( 6, 2 ), ( 7, 4 ), ( 1, 4 ) and connect the points in order to create a parallelogram. What is the perimeter of the parallelogram? Is the perimeter a rational or irrational number? How do you know?)

Answers

Answer: 16, rational

Step-by-step explanation: because its a real number

Which Earth feature was created in a similar way to how much of the solar system was formed? A. Meteor Crater B. the Grand Canyon C. the Petrified National Forest D. Half Dome in Yosemite National Park

Answers

Answer: i think its b.

Step-by-step explanation:

Which of the matrixes can be added together? Choose all that apply.

Answers

Answer: A and C

Both matrices are 1 x 4 matrices. This notation says there is 1 row and 4 columns. The number of rows must match up, as well as the number of columns, in order for matrix addition to be possible. This is so the corresponding elements pair up and add together. For instance, the 5 and -2 pair up and add together for matrices A and C.

Griffin ordered a pair of sneakers online. He had a $22 credit that he applied toward the purchase, and then he used a credit card to pay for the rest of the cost. If the shoes cost $65, how much did Griffin charge to his credit card when he bought the sneakers?

Answers

Answer:$43

Step-by-step explanation:

65 -22 =43


Plsss he helppp fasttttt

Answers

Answer:

c d

Step-by-step explanation:

no explanation

take out a minus from first two fractions

D
Explenation: they are negative in the first one and just listen to me look distribute the negative at the front and makes the fractions negative

Katherine earned 84, 92, 84, 75, and 70 on her first 5 tests.
What is the minimum grade Katherine needs to earn on the next test to have a mean of 84?

Answers

Answer:

D

Step-by-step explanation:

Okay, so the best way to find is too plug in the answer choices.

to find the mean you: add all the numbers up and then, you divide it by however many numbers there are.

minimum meaning exactly 84 no greater!

Start with letter choice A

84+92+84+75+70+81=486

486/6=81 which is  below minimum

Next is letter choice B

84+92+84+75+70+84=489

84/6=81.5 which is below minimum

Next is letter choice C

84+92+84+75+70+95=500

500/6=83.3 which is below minimum

Next is letter choice D

84+92+84+75+70+99=504

504/6=84 which is the correct answer

pls mark me brainliest

HELP ME PLEASE!!! URGENT!!

Answers

Answer:

False

Step-by-step explanation:

See how the bottom of the line F(x) just drops off? That means there isn't an even decline, meaning even the inverse would have an uneven slope.

This line cannot be a function, no matter which way you turn it.

Answer:

false

Step-by-step explanation:

f is  on the Opisite side.

All natural numbers excluding 1 and prime numbers are best described as?
___________________

Answers

Answer:

Composite Numbers.

joy and Peter were each walking along a straight road. Joy walked 1800 meters in 1/3 hour. Peter walked 1/2 of joy's distance in 1/5 hour.
1. What was joy's average walking speed
2. How much did Peter walk in 1/5 hour
3. What is peter's average walking speed
4. Joy walks how much meters/hours faster, slower, or as fast as

Answers

The answer is 3 because it says how much joy walked and not Peter , you’re trying to figure out what’s peters average walking speed

Find the area of the semicircle.

Answers

The answer is 39.27

Please answer correctly !!!!!!!!! Will mark brainliest !!!!!!!!!!!!

Answers

Answer: 33

Step-by-step explanation:

there ages m and b.

m = b + 12

m - 17 = 4(b - 17) m = 4b - 51

3b = 63

b = 21

m = 21 + 12 = 33

Hope this helps :)

Answer: Michael is 33 years old.

Step-by-step explantation: Okay, think of it like this, Michael's age for right now is x since we still do not know Michael's age. Pay close attention to the first sentence. What does it say?  m = b + 2. Now pay attention to the second sentence. What does it say? m - 17 = 4 ⋅ (b = 17). Now, let's move on to the system of equations. As you can see here, the system of equations is

{m = b + 12

{m - 17  = 4 ⋅ (b - 17}.

Now for more equations.

m - 17 = 4b - 68

m = 4b - 68 + 17

m = 4b - 51

m - 4b = -51

-

b + 12 - 4b = -51, through the substitution from the first equation we saw

-3b = -51 - 12

3b = 51 + 12

b = 51/3 + 12/3

b = 17 + 4

b = 21

-

m = 33.

A line passed through the origin and has a slope of 1/2.

Answers

Answer:

D. (2, 1)

Step-by-step explanation:

slope = 1/2

y-intercept = 0

Equation: y = 1/2 x

Choice A:

x = 0

y = (1/2)(0) = 0

Choice A does not work.

Choice B:

x = 1/2

y = (1/2)(1/2) = 1/4

Choice B does not work.

Choice C:

x = 1

y = (1/2)(1) = 1/2

Choice C does not work.

Choice D:

x = 2

y = (1/2)(1) = 1

Choice D works.

Answer: D. (2, 1)

An article is purchased for Rs.5000 andsold for Rs.4500.Find loss percent

Answers

Answer:

10% loss

Step-by-step explanation:

the difference = Rs. 5000 -Rs. 4500 = Rs. 500

Rs. 500/Rs. 5000*100%= 10%

Since it was reduction in price, it is 10% loss

PLEEEEEEEEEASE HEEEEEEEEELP

In a large harbour there are many docks, labeled alphabetically, which are built side by side in a straight line. Dock A and Dock K are located 2.3 km apart along this line. There is a Dock L on an island outside the harbour such that is 45° and is 64°. How far is the island dock away from each of the other two docks?

Answers

Answer:

distance island dock to Dock A = 4.99 km

distance island dock to Dock K = 6.35 km

Step-by-step explanation:

Always make a scetch to visualize the situation.

You need to construct two triangle both with a streight angle, so you can use Pythagoras to calculate the unknown distances between the island dock L, and each of the other two docks A an K.

I chose to introduce an extra letter, the letter C. In total you have the letters A K L and the letter C.

The letter C has a streight angle of 90° between ACL and it has the same streight angle of 90° with KCL. It is crucial that you see that the distance of LC is exactly the same in triangle LAC and that LC has exactly the same distance in the other triangleLKC.

The distance between AK = 2.3 km.

I define the distance between K and point C as 2.3 + x, because the distance x is unknown.

KC = 2.3 + x

Further more, when you make a picture, you can see that the distance between A and point C = x.

From such a picture, it would show clearly, that K is further away in respect to L then point A. From the picture it would be clear that the angle of LKC is smaller then the angle of LAC, so LKC = 45° and LAC = 64°.

Because angle LKC = 45° and we choose C to have an angle of 90°, the TRIANGLE LKC must be a special triangle... In any triangle, the sum of the three angles together, must add up to 180° .

If that is true, then we have 45 + 90 + 45 (because that adds up to 180). Now that means triangle LKC must have two equal sides (because of the same angels of 45° ).

So we know the distance KC = LC and we already defined KC = 2.3 + x.

Now we know enough to solve the problem.

AK = 2.3 km

angle of LKC = 45°

angle of LAC = 64°

AC = x

KC = 2.3 + x

LC = KC

LC = 2.3 + x

Try to calculate the distance x by using tan. After that you can use Pythagoras to find the other distances.

tan(LKC) = ( LC ) / ( KC )

tan(LKC) = ( x+2.3 ) / ( x+2.3 )

That is not helpful. Let's try the other triangle...

tan(LAC) = LC / AC

tan(LAC) = ( x+2.3 ) / x

tan(64) = ( x+2.3 ) / x

Solve the equation which means you try to find the value for x.

x * tan(64) = ( x+2.3 )

tan(64) * x -x = 2.3

tan(64) * x - 1* x = 2.3

Try to get x outside of the braquets...

x* ( tan(64) - 1 ) = 2.3

x* (2.0503038415793 - 1 ) = 2.3

1.0503038415793 * x = 2.3

x = 2.3 / 1.0503038415793

x = 2.19

Now use Pythagoras a² + b² = c² in triangle LAC to find distance LA.

LA² = AC² + LC²

AC = x = 2.19

LC = 2.3 + x = 4.39

LA² = 2.19² + 4.39²

LA = SQRT( 4.79 + 20.16 )

LA = SQRT( 24.95 )

LA = 4.99 km

Now use Pythagoras a² + b² = c² in triangle LKC to find distance LK.

LK² = KC² + LC²

KC = 2.3 + x = 4.39

LC = 2.3 + x = 4.39

LK² = 4.39² + 4.39²

LK = SQRT( 20.16 + 20.16 )

LK = SQRT( 40.32 )

LK = 6.35 km

Other Questions
why do the statues easter island appear damaged? Find the value of X in the triangle shown below Find the least common denominator for these fractions. 5/7 and 4/6 The new Vice President for Operations at The Fresh Connection is looking to speed up setup times using SMED. The first setup using the new methods was successfully completed within 75 minutes. This was projected to improve according to a learning rate of 94 percent. On average, how long should it take to complete the first 5 setups Tu nas pas de cousins? . How many times higher could an astronaut jump on the Moon than on Earth if her takeoff speed is the same in both locations (gravitational acceleration on the Moon is about on-sixth of that on Earth)?1) On the Moon, the jump height is about 3 times higher than on Earth.2) On the Moon, the jump height is about 6 times higher than on Earth.3) On the Moon, the jump height is about 36 times higher than on Earth.4) On the Moon, the jump height is more than on Earth by about a times. find the coordinates of H' after a reflection across the parallel lines; first across y = -2 and then across y = 2. Answer in form (a,b). Part 7a Vaping is the inhaling of a vapor created by an electronic cigarette or other device. True of false A plumber used 2.895 m of pipe during a job. How much pipe did he use rounded to 1 decimal place? b) The ratio of boys to girls in a school is 12:25. If there are 120 boys. (i) how many girls are in the school? (ii) what is the total number of boys and girls in the school? The linear combination method is applied to a system of equations as shown. 4(.25x + .5y = 3.75) x + 2y = 15 One-fourth(4x 8y = 12) x 2y = 3 2x = 18 What is the solution of the system of equations? a(1,2) b(3,9) c(5,5) d(9,3) (-3)^(-2) please help what is the antecedent of the interrogative pronoun What? helppp me plzz?????????? What type of propaganda in nonfiction only gives one side of the story andeliminates important information?A. Glittering GeneralitiesB. BandwagonC. TestimonialD. Framing The following account titles were drawn from the general ledger of Holt Food Supplies, Incorporated (HFSI): Computers, Operating Expenses, Rent Revenue, Building, Cash, Notes Payable, Land, Utilities Payable, Utilities Expense, Trucks, Gasoline Expense, Retained Earnings, Supplies, Accounts Payable, Office Furniture, Salaries Expense, Common Stock, Service Revenue, Interest Expense, Dividends, Supplies Expense. Required A. List each account title under the element of the accounting equation to which it belongs. B. Will all businesses have the same number of accounts? How much work will it take to lift a 2-kg pair of hiking boots 2 meters off theground and onto a shelf in your closet?O A. 2.45 JB. 4JC. 39.2 JD. 20 J A ____________ is a set of commands which can be run by calling it by name. (Phyton) A circle has a radius of \blue{10}10start color #6495ed, 10, end color #6495ed. An arc in this circle has a central angle of 72^\circ72 72, degrees. What is the length of the arc? Either enter an exact answer in terms of \pipi or use 3.143.143, point, 14 for \pipi and enter your answer as a decimal. What is the feeling of wanting to know what happens next within a story?O A. Rising actionB. SuspenseC. TensionD. Exposition